You are on page 1of 320
Unit-1 Sequences of Real Numbers 1. INTRODUCTION (SEQUENCES OF REAL NUMBERS) By a sequence, we mean an arrangement of numbers in a definite order according to some rule. We denote the terms of a sequence by a,, a, a,, ..., et€., the subscript denotes the position of the term. In view of the above a sequence in the set X can be regarded as a mapping of a function f : N —> X defined by f(r) =a, vine N Domain of fis a set of natural numbers or some subset of it denoting the position of term. If its range denoting the value of terms is a subset of R real numbers then it is called a real sequence. ‘A sequence is either finite or infinite depending upon the number of terms in a sequence. We should not expect that its terms will be necessarily given by a specific formula However, we expect a theoretical scheme of rule for generating the terms. Sequence following certain pattems are more often calied progressions. In progressions, we note that each term except the first progresses in a definite manner. A sequence of real numbers is a function whose domain is a set of the form {n < Z| n 2 mh where m is usually 0 or 1. Thus, a sequence is a function f: N — R. Thus a sequence can be denoted by f(m), f(m + 1). f(m + 2), where a yer Usually, we will denote such a sequence by {a}”,, oF {a,, a, fi). If m = 1, we may use the notation (a), Definition : Every function defined from the set N of natural numbers to a non-empty set X is called a sequence. Mathematics (Sequences of Real Numbers) 2. REAL SEQUENCE Every function defined from the set N of natural numbers to a non-emply subset X of the sel of real numbers R is called a real sequence. denoted by f : N > R. Thus the real sequence f is set of all ordered paits {n, f(n)} | (n= 1, 2, 3, ...) Le., set of all pairs (n, f(a) with n a positive integer. N XcR IFf NR is a sequence, then for each n ¢ N, {(n) is @ real number. It is conventional to write {(n) as {,, Notations: Since the domain of a sequence is always the same (the set of positive integers) a sequence may be written as {{(n)} instead of (n, f(n)). 1 Example. The sequence {1, 1/2, 1/3, 1/4, 1/5, ...} is written as ft This sequence can be thought of as an ordinary function f(n) = 1/n. Example. Consider the sequence given by a, = (-1)" for n > 0. The terms of the sequence look like, (1,1, 1, -1, 1, }}. Note that the function has domain }¥ but the range is {-1, 1}. nel Example. Consider the sequence a, = cos {S. 1c N, The first terms in the sequence is 083 * cos 60° = ; and the sequence looks like Note that the function takes on only a finite number of values, but the sequence has an infinite number of elements. Example. if a, = n', n < N, the sequence is aw2.3"" Example. Consider the sequence b, = ( a +1 N, This is the sequence BN) Representation of A Sequence The real numbers x,, X,,..., X, ~. are called the terms or elements of the sequence. x, is called the first term, x, the second term, ..., x, the nth term of the sequence (x). It is denoted by (x,, XX. Ky on) OF) OF HK). € Eduncle Mathematics (Sequences of Real Numbers) Example. ao Ghar) W. (+CV)=(0.2.6,.2 ..). Gi). (0?) = (1 4, 9, 16....). Ww. (CN)=(-41-41..). (v). A sequence (a,) may be defined by a recursion formula : a. = 28, 8, =1. a, - 2a,-VZ1- al ae fia, = V2.J2 Here the terms of the sequence are 1, /2,/2V2.... Eduncle Numbers) 3. RANGE OF A SEQUENCE The set of all the distinct elements of a sequence is called the range set of the given sequence. Example. The range sets of the sequences given in Example are respectively 1 0 f. Py } @ 02 Gi) 1,4, 9, Ww) 1,9 @ — t2f2V2,..4 Note : The range set of a sequence may be finite or infinite but the sequence has always an infinite number of elements, Particular Sequences (a) Constant Sequence : If the n® term of the sequence is constant i.e. 9, = then the sequence obtained (c, ¢, ¢, ...) is known as constant sequence. Example. (5,5, 5,5, >=) (0,0, 0, 0, >=) (b) Identity Sequence : if the n® term of the sequence is a, = n, then the obtained sequence is called Identity sequence. Example. : (1, 2, 3, Equal Sequence eR Vn, Two sequences and are equal, if their n* terms are equal. ie a, € Eduncle 4, BOUNDED AND MONOTONE SEQUENCES Mathematics (Sequences of Real Numbers) Bounded Sequence Bounded above : A sequence (a,) is said to be bound above, if 3 a real number K sit. a, <1 vine N 2 (1+ (AY) = ©, 2, 0, 2, ...) is bounded, ‘Supremum and Inflmum of a Sequence The minimum value of the upper bounds of sequence is known as Supremum or least upper bound (lub) of a sequence. The greatest of the lower bounds of a sequence is known as the infimum or greatest lower bound (glb) of a sequence, Examples 1. The sequence {n*} is bounded below: n? > 0 v n ¢ N but not bounded above; 2. The sequence {-n} is bounded above: -n < 0 ¥ n « N but not bounded below; 3. The sequence {(-1)' + 1} is bounded : (1p + 11< 2 Vine W. € Eduncle Infinitely large sequences represent an important subset of unbounded sequences. Definition : A sequence {a} is called infinitely large if VK ¢ R 3 n, € N such that jal> Ky nen, As an example, we show that the sequence {(—1)" n°} is infinitely large. Indeed, for any number K, we can find n, such that |(—1)" n'| > K vn 2 n,. (Sequences of Real Numbers) To this end, we solve the inequality nt > K, and n > YK. t Let n, = [VK] + 1, where [cl is the integer part of c. Then for n> n, we obtain nen >¥k = n>K = |p n'[>K. From above Definition it follows that any infinitely large sequence is unbounded. However, the converse is not true: there exist unbounded sequences that are not infinitely large. For example, such is the sequence {(1 — (-1)))n). Definition. A sequence {a} is called infinitely small if that is for any © > 0 there exists n, such that lace Vnzn,. For example, the sequence {q*) for jal < 1 is infinitely small, Indeed, for any ¢ > 0 let us find n, such that iq’ < © ¥ 2 n.. To this end, we solve the inequality Iq"| < «, assuming that 0 < ¢ < 1 (for ©2 1, this inequality is clearly true for any n < N): Ine _ [ine] ing’ ™~ |injal} Where Ine < 0, and In [ql < 0, since e < 1, and [ql < 1. Thus, for n 2 n, we have nin(iql) n> Ine ined The fact that {a,} is not infinitely small means the following there exists ¢, > 0 such that for any les nena Me cine = lof ce ne N there exists k, > n with Unbounded Sequence Sequence which is either unbounded above or below is called an unbounded sequence. Monotone Sequences Monotonically Increasing : increasing, if a, 2a, Vn ¢ N. Monotonically Decreasing : A sequence (a,) = (a,, @,, 2 decreasing, if a,., a,,, for all n. It is called strictly increasing if a, < a,,, for all n, and strictly decreasing if a, > a,,, for all n A sequence {a} of real numbers is called eventually non-decreasing if there exists @ natural number N such that a, < a,,, for alln > N, and it is called eventually non-increasing if there exists a natural number N such that a, > a,,, for alln = N. We make analogous definitions of “eventually strictly increasing” and “eventually strictly decreasing.” € Eduncle 5. LIMIT POINT OF A SEQUENCE Mathematics (Sequences of Real Numbers) Let {a,} be any sequence and a « ® we say a is limit point of {a} if every neighbourhood of a contains infinite members of the sequence {a}. i.e. for any 5 > 0 a, € (w- 8, a + 8) for infinite values of n. Ex. (0. for any 5 > 0 ae (1-5,-148) Vn=2k-41,k= 1,2... 1 is a limit point. a, = 1. for any § > 0 ae (1-8, 148) ¥n= AWKET Zw 2 1 is a limit point {a,} has two limit points {-1, 1}. 2; n=1or prime (a= if pin and p is the least prime doing so. a2 for any § > 0 a, < (2-8, 248) ¥n=1 oF prime 2 is a limit point. Let p be any prime for any 5 > 0 ae (@-8 pts ¥N= PL KET 2 pis the limit point of a,, Hence, every prime no. is a limit point of (a, As set of prime no. are infinite {a,} has infinite no. of limit points. Theorem : Every limit point of the range set of a sequence is limit point of a sequence. Solution : Let S be range set of sequence {a}. ie S = range of {a} Leta e S' for > 0 (a ¢, @ + 6) M SMa} has infinite no. of points Let qc (a=, a4) Sila} = gela-sa+sandges As a«S => 4 =a, forsomek e N So a, ¢ (a-s, a+ 8) Hence, (a ~ e, a + £) contains infinite no. of terms of sequence a.is limit point of sequence {a} Proved Remark : Areal no. is a limit point of sequence < it appears in the sequence infinite many times rit is the limit point of the range set. Eduncle Math 6. BOLZANO-WEIERSTRASS THEOREM Theorem : Every bounded sequence has a limit point. Proof : Let (a,) be a bounded sequence Let S = {a : ne N) be its range. Since the sequence is bounded, therefore, its range S is also bounded. Case I. Let S be a finite set Then there must exist at least one element a ¢ $ such that a, = a for an infinite number of values of n. For any © > 0, the nbd. (a - ¢, a + €) of a, contains a, = a, for an infinite number of values of n. Therefore, a is a limit point of ¢a,. Case Il, Let S be an infinite set, The range S being an infinite bounded set has a limit point, say p, So each nbd (p —«, p + «) of p contains an infinite number of elements of S$ ie, a, (pe, p +6) for an infinite number of values of n. Hence p is a limit point of ¢a,). Remark ‘An unbounded sequence may or may not have a limit point. Counter example, Since a, = n is an unbounded sequence with no limit point and a,= 1, if nis even; ,= n, ifn is odd is an unbounded sequence with a limit point 1 € Eduncle 7. LIMIT OF A SEQUENCE (Sequences of Real Numbers) Definition A sequence < a, > is said to have a limit ‘I’ if for sufficiently large values of n, | a,—I | can be made as smail as we please. 1 is the limit of a sequence, if for given « > 0 3n, < N st la-Li O and p< R 4 Lim m=0,when|ri<1andre R 5. Lim 1, when > Q andre R 1 je and let us show that lim 9, Example : Let a, 1 Solution : Given an ¢ > 0, let us choose a N such that 77 < ¢. Now, ifn > N, then we have la,-9 5 sic aN a] which is exactly what we needed to show to conclude that lim a, = Example : Let a, = (2n + 1)/(1 — 3n), and let L = -2/3. Let us show that L = lim a, Solution : indeed, if ¢ > 0 is given, we must find a N, such thet if n > .N then |a, + (2/3) < «. Let us examine the quantity Ja, + 2/3], Maybe we can make some estimates on it, in such @ way that it becomes clear how to find the natural number N. la, + (203) € Eduncle Mathematics (Sequences of Real Numbers) 1" Ble ” a/4 for all n > 1. Therefore, if N is an integer for which N > 1/e, then la, + 2/3) < Tin < IN < 5, whenever n > N, as desired. Example: Let a, = 1/vh, and let us show that lim a, = 0. Solution : Given an c > 0, we must find an integer N that satisfies the requirements of the Gefinition. It's a tittle trickier this time to choose this N. Consider the positive number <*. We know, that there exists a natural number N such that 1/N < 6’, Now, ifn >, then VF =, bees which shows that 0 = lim 1/f. Theorem : Prove that a bounded sequence with a unique limit point is convergent. Proof : Let | be the unique limit point of a bounded sequence (a,). Then for any « > 0. a, € (I, 1+ £) for infinitely many values of n. We show that there exists only finitely many values of n, say m,, m, Mm, such that a, 2, + Gq, do not belong to (I~ «, 1 + s) reason being the infinitely many terms of the given sequence not belonging to (I - «, 1 + +) will have a limit point other then 1 (by Bolzano-Welerstrass Theorem), which is a contradiction. let om-1 ‘ax {m,, m,, .. , m,). It follows that ae Q-el+s) forall n2m ie la-llm. Hence (a,) is convergent to I. Mathematics (Sequences of Real Numbers) 8. LIMIT SUPERIOR AND LIMIT INFERIOR OF SEQUENCE Let (@,) be @ bounded sequence then the sequence has the least and the greatest limit point. The least limit point of {a,} is called limit infinite of {a} and is denoted by Lt inf a,. The greatest limit point {a,) is called limit superior of {a,} and is denoted by Lt sup a, Note : 1. If {a,} is unbounded above the Lt sup a, = © If {a} is unbounded below then Lt inf @,= -w, 2. Since the the greatest limit point of sequence {a,} > the least limit point of sequence {a,) Lt sup a, > Lt inf a, Ex. @ a= (4p. Set of limit point = {-1, 1} Lt sup a, =1 and Lt inf a, 1 @ a=5 Set of limit point = {0} (Lt sup a, =0= Lt inf a,. Wi) a= ty ‘As sequence is neither bounded above nor bounded below. Lt sup a,=42 and Lt inf a, = Note : () If sequence is bounded below then Lt inf a, infimum of set of limit point of sequence a,,. (i) If sequence is bounded above then supremum of set of limit point of sequence a, (ii) if sequence is bounded then Lt sup a,= supremum of set of limit point of sequence a,, Lt. inf a, = infimum of set of limit points of sequence a,, Classification of Sequence 1, Asequence is said to be convergent <> Lt sup a, 2. A. sequence is said to be divergent <> Lt sup a, 3. A sequence is said to be oscillatory => inf a, (finitely) inf a, (infinitely) (Lt sup a, + Lt inf a,, € Eduncle Important ‘A sequence of purely irrational number can converge to a rational number And, @ sequence of purely rational number can converge to a irrational number. let ae Qe for any ne N Mathematics (Sequences of Real Numbers) x, e(e-tart)noia then [x,- al < i (1) Any By Archimedean property ; for any « > 0 3m Ni such that wal) i al<— O there is an integer N > 0 such that if k > N then [a, — L] < ©. The number L is called the limit of the sequence. If (a) converges to L we will write lim a, = Lor simply a, ~ L. If a sequence does not converge, then we say that it elther diverge or oscillate. Note that the N in the definition depends on the « that we were given. If you change the value of © then you may have to “recalculate” N. Consider the sequence a, = =, nc N. Now, if we look at the values that the sequence takes ® 123 4 Lee Er . ani? 1 05 2 05 3 0.375 4 0.25 $ 0.15625, 6 0.09375 7 0.0546875 8 0.03125 9 0.0175781 10_| 0.00976562 We might think that the terms are gelting smaller and smaller so maybe the limit of this sequence ‘would be 0. Let’s take @ look and compare how N would vary as c varies. Let's start with some simple small numbers and let « be 0.1, 0.01, 0.001, and 0.0001, and 0.00001. For & = 0.1, we need to find an integer N so that <01 IS ie Look in the table of values here and we see that for N = 6 we have satisfied the above condition. Following this we get the foliowing by using a calculator or a computer algebra system: N Fra 0 implies n> Sines [<0 N > 9 implies ii-9 < 0.01 € Eduncle Mathematics (Sequences of Real Numbers) 3-4) < 0.0001 N > 18 implies N > 22 implies Fs A sequence is said to be convergent if its limit is finite. lim, Definition : A sequence converges to any number Liff for given 2 > 0 3M, €N (depending on «) la,-Iln Divergent Sequence ‘A sequence is said to be divergent if tim a, OF & Definition : A sequence is sald to be divergent if and only if for any k > 0 3 a no. n, 8. a> k¥ n> ny. For Example, (). The sequence {a,} = {n} = (1, 2, 3, 4, 5, 6, 7, ...} diverges since its limit is infinity (.. (WD. The sequence {a} = (n'} = {1, 4, 9, 16, 25, 36, 49, ...} diverges Gi). Vi = C1" This sequence diverges whereas the sequence is bounded : -1 » but Inn = Inf ~ 1) = IN In 1-0. n= (v). The sequence {sin(nn/2)},., diverges because the sequence Is {1,0,= 1,0, 1, 0, and hence it does not converge to any number. Oscillatory Sequence The sequence which is neither convergent nor divergent is said to be an oscillatory sequence. Le., Sequences that tend to nowhere are always oscillating sequences. For ex. (i. (- 17 + nowhere (i). (- 2" -+ nowhere We remark that an unbounded sequence that does not diverge to ~ or ~ > oscillates infinity. For example, the sequences {1p 4G 1y nh {Coy} are all unbounded and oscillate infinitely. Theorem : Every convergent sequence is bounded. et Us suppose that (a,) be a convergent sequence and € Eduncle Mathematics (Sequences of Real Numbers) Let = 1. Then there exists a positive integer m such that la-ti<1 vnzm = I-t j (Le., for infinitely many values of j). => la-U< a2 forn>j. (2) Let N > max{ i, |} Since N > i, so by (1), law - 1) j, so by (2), la, — 11 . Converse of this theorem also doesn’t hold, such as a,= 1, ifn is even; a, = n, if n is odd is a sequence with a unique limit point 1 but it does not converge to 1 Theorem : A necessary and sufficient condition for the convergent of a monotonic sequence is that it is bounded, Proof : he condition is necessary (=) Let (a,) be a monotonic and convergent sequence. We know that every convergent sequence is bounded. Hence (a,) is bounded, The condition is sufficient (=) Let (a,) be @ monotonic and bounded sequence. Since (a,} is monotonic, we may suppose that (a,) is monotonically increasing. We shall prove that (a,) is convergent. Let $ = {a, ; n ¢ N} be the range of (a). Then S is bounded above, since the sequence (a,) is bounded above By order completeness property, $ has the least upper bound, Let p = Lu.b. S. We shall show that (a,) converges to p. Let « > 0 be any number. Since P—« p ~e, € Eduncle Since (a,) is monotonically increasing, therefore a,20, Ynem ora 28,>p-e ¥n2m or a>p-evn2m of Also p=supS ->a, 0 Since u-eu-e Since a, is monotonically increasing azajru-e yam => @>u-e vnem Al) Also, u is least upper bound of {a,} asu vneN = asute vaom (2) From (1) & (2) u-e la-ul 1, if 1 If the sequence (a,), where a, -Weds 3 tothe ad converges then what is the value of lim 9,7 which Is a geometric progression with common ratio 3 € Eduncle Finihentn (Genvanoen oth | Numbers) lime = 0, fir t< 1. Hence (,) is convergent and lim a, Ex. The sequence (x,) is bounded and monotone where x, = /2 and x,,, = 2x, , then it converges 102. Sol. Observe that x, > x,, Since x?,,-x? = 2(K, - x,,), by induction (x,) is increasing. It can be observed again by induction that x, = 2. Given x, = J2, X= lim, = then lim x,.. aN = @-2F a = 2)=0 620.2 6>0 ene 2 gyuuun 8 € Eduncle Mathematics (Sequences of Real Numbers) 10. THE ALGEBRA OF CONVERGENT SEQUENCES Theorem, If the sequence {a} converges to L and ¢ ¢ R, then the sequence (ca,) converges to cl; ie, c lim a, Proof. Let's assume that ¢ # 0, since the result is trivial if ¢ = 0. Let © > 0. Since {a,) converges to L, we know that there is ann < N so that if n > N. <5. el Thus, for n > N we then have that ea, -cL|= ja. -U| which is what we needed to prove. Theorem. if the sequence {a,} converges to L and {b,} converges to M, then the sequence {@, +b} converges to L + M; ie, lim(a, +b.) = lim a, + lim b, Proof. Let e > 0. We need to find an N ¢ R so that if n > N I@,+b)-C+MI N, then ja, L{-< 5 and ifn > N, then Ib, - Ml} < 5 Thus, if n > mex(N,, Nj} then I(a, +b) -(L + M)) < 19, U1 + o,- M) < 5+ e 2 Theorem. if the sequence {a} converges to L and (b,} converges to M, then the sequence {a,b} converges to L- M;ie., lim (a,b) = lim a, - lim b,, The trick with the inequalities here is to look at the inequality Ja,b, — LM] = Ja,b, ~ aM + aM — LM 0. Then there is a constant K > 0 such that la,! < K for all n. Since {b,) is convergent, for the given ¢ there is an integer N, < N so that If > N, then |b, ~ Mj < ze Also, since (a) is convergent there is ar teger N, « N so that & lfm > N, then la, ~ UI < 2a; Thus if N = maxiN,, Nl} then ifn > N € Eduncle ta,b, — LM| s fa,{b, — MJ + IMI Ja, ~ LI} Mathematics (Sequences of Real Numbers) & e ele SKE IM Tay Ste Lemma. (limits preserve inequalities) If the sequence {a} converges to L and {b,} converges to M and if a, < b, for all n 2 m, then L0- That is, Ls M. OR Let if possible L>M. Since lim a, = L, therefore for e = (L-M)/2, there exists n, such that | @, - L | < (L-M)/2 for all Le. L = (LMY2 n, Similarly, for b,, there exists n, such that | b, ~ M | < (L-M)2 for all n 2 n, ie. M- (L¢M)2 , Le, (-L¥3M)2 M, b,< @,, @ contradiction Hence L 0. 1 Proof. Let e = SILI > 0. Since {a,} converges to L, there is an N < N so that if n> N then fa, - U1 < 1Ly2. Now if n > N we must have that la,| = ILY2. If not then the triangle inequality would imply Lt tlh W=tL-a,tatsib-altlal< 3 +o = Now we set memati Then clearly m > 0 and ja,| > m for all n < N. Theorem. if the sequence {a,} converges to L, if a, + 0 for all n < N, and if L + 0, then the IL. [24 sequence {1/a,} converges to T/L; ie, lim 1/a, = V/lim 4. Proof, Let ¢> 0. By Lemma there is an, m > 0 such that |a,| > m for all n. Since {a,} is ‘convergent there is an integer N < N so that ifn > NiL—al < e-mill. € Eduncle Then forn > N 1 a|_fbe-U Jot a, U . fal mit] Mathematics (Sequences of Real Numbers) Theorem. Suppose that the sequence {b,} converges to M and if {a,) converges to L. If b, + 0 Theorem : Let {a} be a sequence converging to a then the sequence {la,|) converges to lal Proof : This one, we break into three cases. If @ > 0, then (applying the definition of lim, a, = a with c = a) there exists M, so that a, > 0 for n > M,, In this case, we have [a,| = a, forn > M, and lal = a, and so [la,|~ all = [a, ~ al. Since there is M, so that ja, — aj < « for'n > M,, we have that |la,| — |all < ¢ for n > M = max(M,, M,), and so lim, ., la,|= lal. If a < 0, then (applying the definition of lim, ,. a, = a with e = lal) there exists M, so that a, < 0 for n > M, In this case, we have la,| = — a, for n > M, and jal = — a, and so |a,| — [all = La, + al = la, ~ al. Since there is M, so that la, ~ al <= for n > M,, we have that lla, ~ [all < « for n > M = max(M,, M)), and so lim, .. la,! = lal If a= 0, then the definition of lim, a, = @ becomes : for every « > 0, there exists M so that la, = 0| = Jal < «for n > M. Since Ila,|I = a,|, we have that the definition of lim,,, = 0 is satisfied without any further work. OR Let « > 0 be given. Since lim a, = a, therefore there exists n, such that la, — al < ¢ for all n > n,, Hence, | la - lal | < la, al <¢ for all n> n, Thus, lim ja,! = jal Note : Converse of the theorem holds only for a = 0. Thus, lim a, = does not hoid in general. For example, if a, = (-1)° , then lim la, iff im Ja,| = 0. Converse 1 but a, does not even converge. Example, Let p > 0 then jim 2 ie 1 1 1 Let © > O and let N= (2) - Then n > N implies that nF > = and hence « > =>. Since => > 0, ts sone ht n> Nite [2 6 Example. Let [al < 1, then lim a* = ‘Suppose a = 0, because lim a" = 0 is clear for a = 0. Since [al < 1, we can write lal where b > 0. By the binomial theorem soll g Te (epy =tend + AODpe eb 24nd >nb, jematics (Sequences of R (by Now consider ¢ > 0 and let N + i u wipe Then if n> N, we have n >= and hence le" - 0] <3 14na, + 5a(n- 1a? > Fn(n- 1)a, 1 Thus, n> za(n— 1)a,’, $0 9,2 < Thus, we have shown that a, < forn 2 2. Thus, lim a, = 0. Example. jim (2°) = 1 for a> 0. Suppose a 2 1 then for = a we have 1 < a!" 1, so that in) 1 Thus, tim 2)” us, 5 tim a 1 lima" lim av = 1 Recall : For a sequence (a,) we write lim a, N such that n > N implies that a, > M. In this case we will say that {a,} diverges to +x. We can make a similar definition for lim a, Of course, we cannot use the previous theorem when deeling with infinite limits. Theorem, Let {a,} and {b,} be sequence such that lim @, = +x and lim b, > 0. Then lim a,b, = =1 o provided for each M > 0 there is a number $0, Proof. Let M > 0. Choose a real number m so that 0 < m < limb, Whether lim b, = 4 oF not, there exists N, so that if'n > N, then b, > m. Since lim a, = +0 there is an N, so that ifn > N, then € Eduncle Mathematies (Sequences of Re M Setting N = max(N,, N,) means that for n > N, 9,b, > om = M. Theorem. For a sequence {a,} of positive real numbers lim a, = +0 if and only if im = 0, a Proof. Let {a,} be a sequence of positive numbers. We need to show If lim a, = +» then fim 2 w(t) and if tim + = 0 then tim a, wa(2) To prove (1) we will suppose that lim a, = +. Let ¢ > 0 and let M = 1/s. Since {a} diverges to +x, there is an N so that ifn > N then a, > M= 1/c. Therefore, if N > n then ¢ — 1/a, > 0, so if n>N then tq Thus, this proves lim 1/a, = To prove (2) suppose that lim 1/a, = 0 and let M > 0, Let e= 1/M. Then since « > 0 there Is an N so that if'n > N then [-deend Since a, > 0 we then know that al M it m>N then o<2 and hence ifn > N then a, > M. This means that lim a, = +» and 2 holds. Theorem. All bounded monotone sequences converge. Proof. Let {a} be a bounded monotonically increasing sequence and let S = {a,;n € N). Since the sequence is bounded, a, < M for some teal number M and for all n « N. This means that the set S is bounded, and thus it has a least upper bound. Let u = lub S and ¢ > 0. Since u = lub S and ¢ > 0, u—< is not an upper bound for S. This means ‘that there must be some N so that a, > u ~ «. Since {a,} is monotonically increasing we have that for all n > Na, 2 a, and hence for all n > N it follows that u-e < a, su. Thus, a, ~ ul < for all n > N. Thus, lim a, = U = lub S. The proof for bounded monotonically decreasing sequences is the same with the greatest lower bound playing the role of the least upper bound. Theorem. Let {a,) be @ sequence of real numbers. (@® If {@} is an unbounded monotonically increasing sequence, then lim a, = +. (If {a} is an unbounded monotonically decreasing sequence, then lim a, =~. Let (@,) be a bounded sequence of real numbers. While il may converge or may not converge, the limiting behavior of {a,} depends only on the “tails” of the sequence, or sets of the form {a, |1n > N}. This leads us to a concept that if a given sequence converges or diverges. € Eduncle Let u, = gibfa, | n > N} = ina, | m > N) and let v, = lub (a, | nm > N} = supfa, | n > N}. We have seen that if lim a, exists, then it must lie in the interval [u,,, v,J. As N increases, the sets fa, | n > N) get smaller, so we have u, Su, Su, Ss. and v, 2 v, 2,2 By the above theorem the limits u = lim, u, and v = lim, ,. v, both exist and us v since u, £¥, for all N. If the limit exists, then u, < lim a, < v, exists or not. Theorem. A monotonic sequence of real numbers (x,} converges if and only if it is bounded. Proof : We already know that a convergent sequence is bounded. Hence we just need to show that a monotone and bounded sequence of real numbers is convergent. ‘Suppose {x,} is a bounded increasing sequence. Let $ denote the non-empty bounded set (x, : 1 ¢ N}. By the completeness axiom S has a least upper bound, and we let x = sup S. We claim that lim x, = x. Given any ¢ > 0, x ~ € is not an upper bound for S. Thus there exists an integer N such that x, > x —«. Furthermore, since {x,} is increasing and x is an upper bound for S we have K-€ N. Hence lim x, = n. In the case when the sequence is decreasing, let x = inf $ and proceed in a similar manner. Lemma (On nested intervals). Let the sequence of the intervals ([a,, b,]} be such that {a,,,, b,,,] [a,b vn e N (such intervals are called nested), and their lengths converge to zero, ie., lim(b, ~ a.) = 0. Then there exists a unique point & that belongs to every interval, ie. < [a,b] vn < N, and ati (Sequences of Real Numbers) &= Jim a, = supfa,) = lim b, =inf{b,). Proof. The sequence {a,} and {b,} are monotonic and bounded: a,|g-nj>0 vn = lim (b, -a, >|e-n]>0. This contradiction completes the proof of the lemma. Example : Determine if the following sequences are monotonic and/or bounded. @ bey © {0} Eduncle 2 ® 8 Solution. @) {- This sequence is @ decreasing sequence (and hence monotonic) because, one > (n+ 1 for every n. Also, since the sequence terms will be either zero or negative this sequence is bounded above. We can use any positive number or zero as the bound, M, however, i's standard to choose the smallest Possible bound if we can and it’s a nice number. So, we'll choose M = 0 since, = <0 for every n This sequence is not bounded below however since we can always get below any potential bound by taking n large enough. Therefore, while the sequence is bounded above it is not bounded below. we can also note that this sequence diverges (to —«). rh, The sequence terms in this sequence alternate between 1 and -1 and so the sequence is neither an increasing sequence or a decreasing sequence. Since the sequence is neither an increasing nor decreasing sequence it is not @ monotonic sequence. It is an osciliatory sequence. The sequence is bounded however since it is bounded above by 1 and bounded below by —1. Again, we can note that this sequence is not divergent, it is oscillatory. ofr, This sequence is a decreasing sequence (and hence monotonic) since, 2 (n+ The terms in this sequence are all positive and so it is bounded below by zero. Also, since the sequence is a decreasing sequence the first sequence term will be the largest and so we can see that the sequence will also be bounded above by 2/25. Therefore, this sequence is bounded. We can also take @ quick jimit and note that this sequence converges and its limit is zero. Example. Determine if the following sequences are monotonic and/or bounded. w {ot { iat aaa Solution. € Eduncle Mathematics (Sequences of R | Numbers) We'll start with the bounded part of this example first and then come back and deal with the increasing/decreasing. First, n is positive and so the sequence terms are all positive. The sequence is therefore bounded below by zero. Likewise each sequence term is the quotient of a number divided by a larger number and 80 is guaranteed to be less than one. The sequence is then bounded above by one. So, this sequence is bounded Now let's think about the monotonic question. First, we will offen make the mistake of assuming that because the denominator is larger the quotient must be decreasing. This will not always be the case and in this case we would be wrong. This sequence is increasing. To determine the increasing/decreasing nature of this sequence we will need to calculate I technique. First consider the following function end its derivative. r 1 = (x)= f= (x) my We can see that the first derivative is always positive and so from calculation we know that the function must then be an increasing function, Notice that, fin)= Therefore because n < n + 1 and {(n) is increasing we can also say that, a nl =1(n) a, {30000 = 13.16 and so the function will be decreasing in this range. So, our sequence will be increasing for 0 5 - We have x, > 3 and if we assume that x, > 3 for some k ¢ W, then x,,, > 5—2= 3. Hence by the principle of mathematical induction, x, > 3 for all n < N. Therefore (x,) is bounded below. Again, x, £25 ~ S(aince x, > 3 for alln © N, (40) => (/ = 20-3) = 0 f= 2or/= 3, But x, > 3 for al ne N, 90/23, Therefore f= 3. 2, (2k)! seat: a ste (20 8) can or eee Solution : The given series is a sum of two series. We will look at each of them separately and hope that the two results can be put together. First consider the series 2 is This is @ series that can be converted to a geometric series, 2 2 1251) Lyre Fle-gLLy) Since q = 1/5 satisfies |ql < 1, this series converges. Now we will look at the series (8)! ae When we see factorial, we immediately think of the Ratio test. ( (2k+7))! —tim( 221) = tin! 3 (2+ 2)! 3k Feta HE) ear [ER an 7 =n aa esaks 24) 1:2-3--(2k) (MEH Since 4 > 1, this series diverges. Since the given series is the sum of a convergent and a divergent series, it is divergent. € Eduncle 11. SUBSEQUENCE (Sequences of Real Numbers) Definition : Let (a,) be any sequence. Let (n,, n,. such that i > j =n, >, Then the sequence (a,,,2,. subsequence of (a). n, ..) be a sequence of positive integers }, written as (a, ), is called a Let {2,);, be @ sequence. When we extract from this sequence only certain elements and drop the remaining ones we obtain a new sequences consisting of an infinite subset of the original sequence. That sequence is called a subsequence and denoted by {4, }i, (Big, yy Qype -») 1S @ SUbsequence of (2, Theorem : If sequence bdd., then every subsequence is bdd and conversely. Proof. Let {a} be any bdd. sequence then 3 m,M « R such that msasMvneN 0) Let {a,,} be any subsequence of {a,} => ms 4, |, therefore, for any ¢ > 0, there exists a positive integer m such that la- lee vn>m. Inpanticular, 1a, -ll< & vn, m. Hence (a,,) converges to | Remark : The converse of the above theorem may not be true. ‘Theorem : Every bounded sequence has a convergent subsequence. Proof : Let is bounded Case 1: When the range of the sequence is finite. If the range of in finite then at least one element ‘a’ repeats infinitely because a sequence has infinite number of terms, if we choose this element ‘a’ again and again then the subsequence formed Is convergent, Case 2 : When the range of the sequence is infinite . For eachnn ¢ N, 3V, € NS Let . forgiven > 03neNst Tee then x, -Uetet ny 7 x tim x,, =! >is convergent. Theorem : if the subsequences {a,,,} and {a,,} of a sequence {a,} converge to the same limit I, then the sequence (a) converges to L. Proof. Let « > 0 be given {a,,.,) converges to 1 => Fore > 0,3 a positive imeger m, such that la, I< e ¥ n= Mm, {a,,,) converges to 1 = Fore > 0,3 positive imeger m, such that a, —llm, Let m = max. {m,, m,}, then |a,,,-U la-ice vnem => {a) converges to € Eduncle Ex. Take the sequence {(-1)'};,. Extract every other member, starting with the first. Then do the ‘same, starting with the second. The sequence in question is (CW¥j4={-00-V4-44.4 If we extract every second number, starting with the first, we get: 4, -1, 1, -1, ..} This subsequence now converges to -1 If we extract every second number, starting with the second, we get: 1. TT This subsequence now converges to 1. . \ . Examples : The sequence ( 7) in Ris a subsequence of the sequence (G i} hs hes (in this case, n, := j?). The sequence 1, 1, 1, 1, ... is a subsequence of 1, 0, 1, 0, 1, ... Take the sequence ( Extract three different subsequences of your choice and look at the convergence behavior of these subsequences. The sequence in question is : VM, which converges to zero. Now let us extract some subsequences: Extracting the even terms yields the subsequence {1/2, 1/4, 116, 1/8, 1/10, which converges to zero. Extracting the odd terms yields the subsequence (1, 113, 5, 77, 19, «) which converges to zero. Extracting every third member yields the sequence {1, 1/4, 1/7, 1/10, 1/13, ..} which converges to zero. Hence, ail three subsequences converge to zero. This is an illustration of a general result: If a 1, 1/2, 13, 1/4, 15,1, sequence converges to a limit L then every subsequence extracted from it will also converge to that limit L * — Onthe other hand, itis possible for a subsequence to be convergent without the sequence 5 a whole being convergent. For example, the sequence 1, 0, 1, 0, 1, ... is not convergent, even though certain subsequences of it (such as 1, 1, 1, ... converges). —<—$—— € Eduncle 12. CAUCHY SEQUENCE Mathematics (Sequences of R Definition. A sequence of real numbers is said to be a Cauchy sequence if for every o> O there exists a natural number N such that for all natural numbers n, m > N, the terms a,, a,, satisfy la, -a,t 0, there exists a positive integer m such that la, — a1 < ¢, whenever n > m. Note: It can be easily shown that the two definitions are equivalent. Most of the authors use the 1# one as definition but in practice we'll often use the latter one. Theorem : Cauchy Sequence is bounded. Proof : Let (a,) be @ Cauchy sequence For & = 1, there exists @ positive integer m such that la,-a,l<1, whenever n > m, or a-1 be a cauchy sequence, then by definition for any © > 03 n, sit 1x, 7%, 1¢ 2/2 vmn>0 (1) Let be the subsequence of and lim X,,= 1, then for !>0 3 n,< Nt. Im- E1€ 02 Wy ony @ Now |x, - LP = 1x,F 1 S(%, 7%) +1G,- 11 < 02+ 62 vn>v, [by (1) and (2)] => |x-ll limx,: Theorem : A sequence X = in R is convergent if and only if it’s Cauchy. Proof : “=>" If x := lim X, then given <> 0 there is a natural number K such that ifn > K then IXn-x1< €/2. Thus, ifn, m > K, then we have Ix, -x,1= 1%, =x) + Kx) Lc - x14, “x1 < 6/2 + g/2 = ¢.Since < > 0 is arbitrary, it follows that is a Cauchy sequence. (Here we used the first definition) “ e=" let X = (x,) be a Cauchy sequence; we will show that X is convergent to some real number. First we observe that the sequence X is bounded. Therefore, by the Bolzano-Weierstrass Theorem, there is a subsequence X’ = of X that converges to some real number x*. We shall compiete the proof by showing that X converges to x* Since X = (x,) is a Cauchy sequence, given > 0 there is a natural number N such that if n, m N then bx.-x,< € Eduncle Since the subsequence X’ set (n,n, ... } such that Kextle . TUS AN Dox ke IK xtllx, xl Hence, X converges to x* Theorem : (Cauchy's Principle of Convergence) A necessary and sufficient condition for a series 5 u, to converge is that to each « > 0, there exists a positive integer m, such that |S, - §,| s © er Proof : Let (S,) be the sequence of partial sums of the series 5 u,. By definition, 5 u, converges © (S,) converges = Toeach c > 0, there exists a positive integer m such that IS,-S,lm (Cauchy's Principle of Convergence for Sequences) jematics (Sequences of R converges to x’, there is a naturai number K>N belonging to the xt. +. + UL <6, for all n> m. © tut. tutu, tut. tu) —W,tu+.. tul m for which a, = 1 and la, - 4,1 = 11 + 11 = 2 <1, a contradiction. Hence ((-1)) is not a Cauchy sequence. 4. Consider the series (that is, infinite sum) s-y4 oa We may view this as the limit of the sequence of partial sums bo es We can show that the limit converges using Theorem by showing that {a} is a Cauchy sequence. Observe that if j, k > N, we definitely have boals ear It may be difficult to get an exact expression for the sum on the right, but it is easy to get an upper bound. “ San Sunt) n= € Eduncle Matai (ico Wt Mra) Sol. Sol. Sol. 1 The reason we used the slightly wasteful inequality, replacing —> by the sum on the right telescopes, and we know itis exactly equal to jj. To sum up, we have shown that when j, k > N, we have balsa Let 0 < a <1 and let (x) satisfy the condition Ix, ~ X,| < a® for all n ¢ N then (x) is cauchy. For all m, ne N with m > n, we have bk, — X1 $ bk, — Kal # Bay — Xyl + oe + gg = Sa" tom + bam (-a™)< Since 0 < a <1, a" > 0 and so Ta Ta given any « > 0, we can choose n, ¢ N such that “—(1 - 0) < &. Hence for all m,n 2 ny we have ix, - x < all ~ 0) < c. Therefore (x,) is a Cauchy sequence. 1 tatatg teeta forall me N then (x) is 1 4.4 4 Wo Wa mM Se eT JeZe2. cen e 0, cao ne W iting n> 2 For all m, ne N with m > n, we have Ix, — XJ = Then for all m,n 2 n, we get x, — x1 < zs &. Consequently (x,) is @ Cauchy sequence in R and hence (x,) is convergent. ‘Show that the sequence (a,) defined by a, = (-1)”n converges. Hf we show that (a,) is a Cauchy sequence, then by Cauchy's General Principle of Convergence, (a,) converges. Let > 0. If n> m, then 1/n < 1/m and 1 Let m be a positive integer > 2/e. Then la,-a,| N, where C is @ constant. 1 Solution. Let = = 5. Since {a,};. is Cauchy, there is 2 positive integer N such that if m,n > 1 N we have la, ~a,! < >. But a, ~ a, is an integer so we must have a, = a, for all n 2 N. Example : What does it mean for @ sequence {a,);., to not be Cauchy? Solution. A sequence {a,};., is not a Cauchy sequence if there is @ real number > 0 such that {for all positive integers N there exist n, m ¢ N such that n, m > N and ja, -a,| <6. Example : Let {a,);, and {b,);,, be two Cauchy sequences. Define c, la, ~ b,). Show that {c,)z. is a Cauchy sequence. Solution. Let e > 0 be given, There exist positive integers N, and N, such that if n, m2 N, and n, m2 N, we have la, ~ a,| < Zand Ib, byl < Let N= N, +N,. ifn, m2>N then Ic, - ¢,1 = lla, -b- la, -b,ll s(@,-b) + (@, - b,)/ 0, since (s,),.,, converges, there exists N © N such that for every n « N with n > N, Is, - s|is less than e. For every k < NV with k 2 N, since n, is at least as large as k, in particular n, = N. Therefore, ka is less than «. Thus (5, ), Jen converges to 5. Next, let (3,),,, be @ Cauchy sequence. For every ¢ > 0, there exists N eN such that for every m,n cl with m>N and with n> N, Is, - s,| is less then «. For every k, / el¥ with k > N and with / > N, then n, > k>N and n,>/>N. Therefore, is less than c. Thus (5, },_., is a Cauchy sequence k € Eduncle ‘Mathematics (Sequences of R Example : if @ subsequence of a Cauchy sequence converges to L, then the full sequence also converges to L. Example : Prove directly from the definition that the sequence | Numbers) is a Cauchy sequence, Solution : Let « > 0 be given. Let N be a positive integer to be chosen. Suppose that n, m2 N. We have Ja ag[=[+3_m*3]_5__lmoo]_ nl laneT amet] n+ ams) _2M42m __ (2+) (2m+1)-2 (2n+D(2m+1)~ (2+ )(2m+1) 1,4 2 Qm+1" 2n+1 (ns 1)am+1) 1 1 < + am+1" 2n+1 2 NAT Choose N so that —2—< ¢. This is N > 2—£. in this case, N+ © 2° * alee for all n, m 2 N. That is, est is Cauchy. Example : Consider a sequence defined recursively by a, = 1 and a,,, = 8, + (-1)" n? for all n N, Show that such a sequence is not @ Cauchy sequence. Does this sequence converge? Solution. We will show that there is an « > 0 such that for all N ¢ 'N there exist m and m such that m, n > N but a, ~a,| > e. Note that la,,, - a,| =n? > 1. Let N € N. Choose m= N +1 and n= N. In this case, la, — a, = N?21 = c. Hence, the given sequence is not a Cauchy sequence. Since every convergent sequence must be Cauchy, the given sequence is divergent, Theorem : (Cauchy's first theorem on limits) 8+) +48) If lim a, =1, then lim pee ru n Proof : Let b,=a,-! or a=l+#b,¥neN. of Then limb, =lim a,-U=t-1=0 lim b, =0. m0) Now 248% +8 7 Bitbe++De using (i) 7 n To show that jim, ( € Eduncle jematics (Sequences of Real Numbers) Let « > 0 be any number. Now lim b = 0 implies that there exists a positive integer m such that € « b-1<5 Vnem > bl< > v nem. oi) Again lim b, = 0 = (b,) is convergent = (b,) is bounded = there exists a number K > 0 such that [b,| 2mK n 2 n 2 © Let p be a positive integer > 2a Then by +b; +n. + Ds 7 ) Now [bas +By.2 ++, 7 $1 Bys 1+ 1Dp a 14+ 1B, D € Eduncle Remark The converse of the above theorem may not be true. For example, (a,) = (-1, 1, —1, 1, ...) is not convergent and a,+a,=0,a,+a,+a,=-1,a, +, +a, +a, =0, etc. ihematics (Sequences of Real Numbers) Now 2*%*= 4%. 6 ifm is even = 1, itis oad. n tet8,) Hence tim (22ut=t2) nd Theorem : (Cauchy's second theorem on limits) but lim a, + 0. If jim, a, = L where a, > 0 Vn ¢ N, then lim (a,, a, ... a)!" = 1. Proof : lim a,=!= lim (og ,) oa{ im a, By Cauchy's first theorem on limits, lim (log @,) = log . yy (amc, Segal, +. +1008, +..+lo9a, »)- ‘agi 1 = im 1 (Froata, a tog! > Jn [oata.e, A Hence im (aa, «a, Corollary : Suppose a,) is a sequence of positive real numbers such that jim %24=a(a>0), Then lim (a,)" =a Proof : Let a, = 1. Then I By Theorem it follows that € Eduncle Mathematics (Sequences of Real Numbers) (3n)! (3n+ 3)! Solution. We have a, = Gn’! (ay iF im (2023) (0 a, ete (n+? * (3m)! . (3n + 3)(3n + 2)(3n+1) 32 rg = iy B+ 3/08 + 2/NYS +11) _ 9 am (istiny Using the Corollary of Theorem we get iim (a,)"" = 27. Example : Prove that if a, = Mn e142). (0+)! " then (a,) converges to 4/e (n+1)(n+2)..dn+n) Solution. Let b, + 80 that (b,)"" Werte tig, (n+ 2yin+ Hen +1)(2n+2) mb eee (ne net) (ne 2)... 20 = im (=) (2n+1(2n+2) 1 (2+1/n)(2+2in) _ 4 G+ny (+n Hence jim (b)!* = Jim a, = 4/e. = lim ‘Some important theorems on limits Theorem : if a,20¥ne Nand jim a, =I, then! 20. Proof : Let, if possible, 1< 0. let e=-1>0. Since lim a, = 1, so there exists @ positive integer m such that la-e-l vo nom => a,-[sla,-N<-l = a<-l+l=0 ¥ nem > <0 ¥ nem, which contradicts the fact that a, > 0 vn. Hence I> 0 Theorem : If (9,) and (b,) be two sequences such that a, a, > b,-a,20 = — lim (b,-a,)20, € Eduncle = limb,-lim a,20 = lim bz lim a. Mathematics (Sequences of Real Numbers) Hence lim a, 0 be given, Since lim b, = l, there exists a positive integer m, such that lace vooven, > l-c l-em, Let m = max (m,, m)}. Then l-ceacl+e and I-c 1b,-Il 1, then lim (a,) = 2 3. if l= 1, then Test faits. Proof : (1) Since I 11 < 1, we can choose @ positive number « so small such that llj+e+#1 of K<1,wherek=([U|+e. sil) Since lim "= 1, there exists a positive integer m such that Ix-yl2ixt-lyD ties (Sequences of Real Numbt hae) = a] ake, (2) We know k" > 0 asin ~~, since O =. Hence lim a, = 0. Proof : (2) Since 1 > 1, we can choose a positive number e such that l-e>1 or k>1,wherek=1-« Since lim 25-1 = |, there exists a positive integer m such that => I-c< m. Pe Vv nem, Putting n = m,m +1, ..., (n — 1) successively and multiplying, we get => lal Nowk>1 > ki’ >a asn—>o, Hence lim a, = =. Proof: (3). Let a,= 1 Vn <1,1,1,...> 31 (constant sequence) (this is a convergent sequence) ‘duncle Numbers) Consider a,=nvn <1.23,4.m 941, > 9 (this is a divergent sequence) Soa 4) yt stb asa a a So in this case we cannot conclude anything about the sequence. Example : Deduce that lim 2* n? = 0. vr (n+ ¥ Solution : Let a, = >; so that a,,, = “= (as 2 yeasty zn 2 oder. Hence, by Theorem im a, = lim 2" n?=0. Example : Show that lim (3° n?) = 0. Solution : Let a, = 3 80 that a, ., = etl Hence jim a, im 3° 1? = 0 Example : Show that for any number x, i x Solution, Let a, = Ty, 80 that a, ,, = Tray Sen 1 nt a, (n+) x nel im %-0<1, xeT Hence lim a,=0 ie, tim a ne Example : Show that m(m—1)(m—2) .. (m=n +9 —<— lim =0,if|x1<1. € Eduncle jermatics (Sequences of Real Numbers) m(m-1)...(m-n+1) x, $0 that nl Solution : Let a, _ m(m=1).. (m= Mim cam (oo! lim, Hence jim a, = 0, Example : Prove that if p > 0, then | wer 0, k being a fixed real number. i . Solution. Let a, = —"\_, so that a,,, = 070 (+ py (spy ar 1 Now im === amt (1 a Te a where 7 5<1 (v 14 p>1asp>0) Hence lim a, = 0 (ry Example : Show that lim “*¥" = 0, for all y. Solution. Let a, = fon , $0 that 2,,, nay 1 ty, Now CON Ene tim 2-2 jim 1*¥20<1, Be Hence jim a,= 0 ie, tim C+¥". 9, Fo ne Unit-2 Series of Real Numbers 1, INTRODUCTION (SERIES OF REAL NUMBERS) A finite series is given by the terms of a finite sequence, added together. For example, we could take the finite sequence (2k +1).2, =(3,5,7...21) Then the corresponding example of a finite series would be given by all of these terms added together, 3+5474..+21 We can write this sum more concisely using sigma notation. We write the capital Greek fetter sigma, and then the rule for the kth tem. Below the sigma we write 'k = 1’. Above the sigma we write the value of k for the last term in the sum, which in this case is 10. So in this case we would have 2k+1=3+5474..421 ‘and in this case the sum of the series is equal to 120 In the easy way, an infinite series is the sum of the terms of an infinite sequence. An example of an infinite sequence is (a1 ,-Gas~} And then the series obtained from this sequence would be With a sum going on forever. Once again we can use sigma notation to express this series. We write down the sigma sign and the rule for the k-th term. But now we put the symboi for infinity above the sigma, to show that we are adding up an infinite number of terms. In this case we would have yi) F234 1 a Key points : A finite series is given by all the terms of a finite sequence, added together. A infinite seties is given by all the terms of an infinite sequence, added together. Definition ‘An expression of the form atatat..¢at. where each a, is @ real number. is called an infinite series of real numbers and is denoted by Xa, o ¥.a,.2, is called the nth term of the series ¥ a, € Eduncle Mathematics (Series of Real Numbers) Example. . is an infinite series. The sequence of partial sums looks like : We saw above that this sequence converges to 2, so Spelim s, ao Example. The number nl, read n factorial, is defined as the product of the first n positive integers; meds Qe (nl) is an important sequence. Its first few terms are 1, 2, 6, 24, 120, 720, By convention, 0! is defined by 0! = 1 © see ET Sequence of Partial Sums of Series Recall : A sequence {a} is a function which assigns a real number a, to each natural number n:1,2,3,4,5, ..., ie, @ sequence is an ordered list of real numbers : ,, 8,, 8, 8, 2, 2 1 Example : {5} generates the sequence 5. € Eduncle Mathematics (Series of Real Numbers) Definition : An infinite series 5a, is the sum of the numbers in the sequence {a}. i. Ee satatatatat... Example: 25 We need a more precise definition of an infinite series 3\a,. Begin by constructing a new sequence of partial sums by letting (This step by step by step process will be called the Sequence of Partial Sums Test for the infinite series a, .) 5,78, +a, 8,8, +a, +2, atatata, a,tata, tat. +a, We can now say that the value of the infinite series is precisely the value of the limit of its sequence of partial sums, i.e. =lim (a, +8, +a; +8, +...+8,) ain Examples of Partial Sums For the sequence 1,1,1,1,.... we have J" 1=N—. Thus , J ,1 is divergent. For the sequence 1,-1,1, we have S, = 1, S, = 0, S, = 1, etc. In general, S, = 1 for N odd and S,= 0 for N even, Thus, >" ,(-1)" is oscillatory. The Harmonic Series If one computes the partial sums for >. <1 Sj =2.93, Spy = 3.40, Shoop = 7.49, Siooo0 *12.09. In fact, S,->% So that Y, a dverges: ~ 1 If one computes the partial sums for law then one obtains S, = 1, s,-$-1 25, S, = 49 36 7196, Sip 21.55.89 21.63, Syyo =1.64 45 = 1.644934068 = i Infact, Dor =<(2) aw

You might also like